Difference between revisions of "2010 AMC 10A Problems/Problem 19"
(→Solution) |
(→Solution) |
||
Line 50: | Line 50: | ||
===Solution 3 (no trig)=== | ===Solution 3 (no trig)=== | ||
+ | <asy> | ||
+ | unitsize(5cm); | ||
+ | defaultpen(linewidth(.8pt)+fontsize(8pt)); | ||
+ | draw((0,0)--(-0.5,0.866)--(-0.414, 1.015)--(0.586,1.015)--(0.6716,0.866)--(0.1716, 0)--cycle); | ||
+ | draw((0,0)--(-0.414, 1.015)--(0.6716,0.866)--cycle); | ||
+ | draw((0.586,1.015)--(0.6716,1.015)--(0.6716,0.866)); | ||
+ | |||
+ | label("$E$",(0,0),SW); | ||
+ | label("$F$",(-0.5,0.866),W); | ||
+ | label("$A$",(-0.414, 1.015),NW); | ||
+ | label("$B$",(0.586,1.015),NE); | ||
+ | label("$C$",(0.6716,0.866),E); | ||
+ | label("$D$",(0.1716, 0),SE); | ||
+ | label("$M$",(0.6716,1.015),NE); | ||
+ | label("1",(-0.25,0.433),SW); | ||
+ | label("r",(-0.457,0.9405),NW); | ||
+ | label("1",(0.086,1.015),N); | ||
+ | label("r",(0.6288,0.9405),NE); | ||
+ | label("1",(0.4216,0.433),SE); | ||
+ | label("r",(0.0858,0),S); | ||
+ | |||
+ | </asy> | ||
Extend <math>AB</math> to point M so that it creates right triangle <math>\triangle AMC</math> where <math>\angle M = 90^\circ</math>. It is given that the hexagon is equiangular, therefore <math>\angle MBC = \frac{360}{6} = 60^\circ</math>. (exterior angles of a polygon add up to 360 <math>^\circ</math>) | Extend <math>AB</math> to point M so that it creates right triangle <math>\triangle AMC</math> where <math>\angle M = 90^\circ</math>. It is given that the hexagon is equiangular, therefore <math>\angle MBC = \frac{360}{6} = 60^\circ</math>. (exterior angles of a polygon add up to 360 <math>^\circ</math>) | ||
Revision as of 04:07, 16 January 2020
Problem
Equiangular hexagon has side lengths and . The area of is of the area of the hexagon. What is the sum of all possible values of ?
Solution
this diagram only shows one possible value of r
Solution 1
It is clear that is an equilateral triangle. From the Law of Cosines on triangle ABC, we get that . Therefore, the area of is .
If we extend , and so that and meet at , and meet at , and and meet at , we find that hexagon is formed by taking equilateral triangle of side length and removing three equilateral triangles, , and , of side length . The area of is therefore
.
Based on the initial conditions,
Simplifying this gives us . By Vieta's Formulas (or Girard identities, or Newton-Girard identities) we know that the sum of the possible value of is .
Solution 2
As above, we find that the area of is .
We also find by the sine triangle area formula that , and thus This simplifies to .
Solution 3 (no trig)
Extend to point M so that it creates right triangle where . It is given that the hexagon is equiangular, therefore . (exterior angles of a polygon add up to 360 )
We can use either Pythagorean theorem or the properties of a triangle to find the length of and . The legs of are and .
Using Pythagorean theorem, we get . We can then follow to solve for . .
Alternatively, we can find the area of . We know that the three smaller triangles: , , and are congruent because of . Therefore one of the smaller triangles accounts for of the total area. The height of the smaller triangle is just so the area is . We can then find the area of the hexagon using .
We can even find the area of and and solve for because the ratio of the areas is to .
~Zeric Hang
See also
2010 AMC 10A (Problems • Answer Key • Resources) | ||
Preceded by Problem 18 |
Followed by Problem 20 | |
1 • 2 • 3 • 4 • 5 • 6 • 7 • 8 • 9 • 10 • 11 • 12 • 13 • 14 • 15 • 16 • 17 • 18 • 19 • 20 • 21 • 22 • 23 • 24 • 25 | ||
All AMC 10 Problems and Solutions |
The problems on this page are copyrighted by the Mathematical Association of America's American Mathematics Competitions.